Merge remote branch 'public/master'
[course.git] / latex / problems / Serway_and_Jewett_4 / problem07.62.tex
1 \begin{problem*}{7.62}
2 A roller coaster car is released from rest at the top of the first
3 rise and then moves freely with negligible friction.  The roller
4 coaster shown in Fig.~P7.62 has a circular loop of radius $R$ in the
5 vertical plane.
6 \Part{a} First, suppose the car barely makes it around the loop; at
7 the top of the loop the riders are upside down and feel weightless.
8 Find the required height of the release point above the bottom of the
9 loop, in terms of $R$.
10 \Part{b} Now assume that the release point is at or above the minimum
11 required height.  Show that the normal force on the car at the bottom
12 of the loop exceeds the normal force at the top by six times the
13 weight of the car.  The normal force on each rider follows the same
14 rule.  Such a large normal force is dangerous and very uncomfortable
15 for the riders.  Roller coasters are therefore not build with circular
16 loops in vertical planes.  Figure P5.24 and the photograph on page 134
17 show two actual designs.
18 \end{problem*} % problem 7.62
19
20 \begin{solution}
21 \Part{a}
22 Because the riders ``feel weightless'' at the top of the loop (point
23 $T$), we will assume that they are in free fall with a centerwards
24 acceleration of $g = v_T^2/R$.  Conserving energy between $T$ and the
25 release point $A$
26 \begin{align}
27  E_A = mgh &= E_T = \frac{1}{2} m v_T^2 + mg(2R) \\
28  h &= \frac{1}{2g}v_T^2 + 2R = \frac{Rg}{2g} + 2R = \ans{2.5 R}
29 \end{align}
30
31 \Part{b}
32 If the release comes from a higher point, there will be some normal
33 force at the top $N_T$ and at the bottom $N_B$.  Summing forces at
34 both points
35 \begin{align}
36  \sum F_{cT} &= mg + N_T = m \frac{v_T^2}{R} \\
37   v_T^2 &= gR + R\frac{N_T}{m} \\
38  \sum F_{cB} &= -mg + N_B = m \frac{v_B^2}{R} \\
39   v_B^2 &= -gR + R\frac{N_B}{m}
40 \end{align}
41 And conserving energy between the top and bottom
42 \begin{align}
43  E_B = \frac{1}{2}mv_B^2 &= E_T = \frac{1}{2}mv_T^2 + mg(2R) \\
44  v_B^2 &= v_T^2 + 4gR = -gR + R\frac{N_B}{m} = gR + R\frac{N_T}{m} + 4gR \\
45  N_B &= N_T + 6mg
46 \end{align}
47 \end{solution}